包除原理

数学Aで次の式を学びました.

\[
\begin{align*}
n(A_1 \cup A_2)=&n(A_1)+n(A_2)\\
&-n(A_1 \cap A_2)
\end{align*}
\]

\[
\begin{align*}
n(A_1 \cup A_2 \cup A_3)=&n(A_1)+n(A_2)+n(A_3)\\
&-n(A_1 \cap A_2)-n(A_1 \cap A_3)-n(A_2 \cap A_3)\\
&+n(A_1 \cap A_2 \cap A_3)
\end{align*}
\]

授業でベン図を用いて理解したことと思います.

この2つの式の流れ的に,

\[
\begin{align*}
&n(A_1 \cup A_2 \cup A_3 \cup A_4)\\
=&~n(A_1)+n(A_2)+n(A_3)+n(A_4)\\
&-n(A_1 \cap A_2)-n(A_1 \cap A_3)-n(A_1 \cap A_4)-n(A_2 \cap A_3)-n(A_2 \cap A_4)-n(A_3 \cap A_4)\\
&+n(A_1 \cap A_2 \cap A_3)+n(A_1 \cap A_2 \cap A_4)+n(A_1 \cap A_3 \cap A_4)+n(A_2 \cap A_3 \cap A_4)\\
&-n(A_1 \cap A_2 \cap A_3 \cap A_4)\\
\end{align*}
\]

などが,そしてさらに一般化して,

\[
n\left(\displaystyle\bigcup_{i}^{n} A_i\right)=\displaystyle\sum_{i}^{n} n(A_i)-\displaystyle\sum_{i<j}^{n}n(A_i \cap A_j)+\displaystyle\sum_{i<j<k}^{n}n(A_i \cap A_j \cap A_k)-\cdots+(-1)^{n-1}\displaystyle\sum^{n}_{i<j<\cdots} n(A_i \cap A_j \cap \cdots )
\]

が成り立ちそうな気がします.

ここで,\(\displaystyle\bigcup_{i}^{n} A_i\)は\[A_1 \cup A_2 \cup ~\cdots~ \cup A_n\]を,\(\displaystyle\sum_{i<j}^{n}n(A_i \cap A_j)\)などは\(1\leq i<j\leq n~(i,j\in \mathbb{N})\)をみたす\(n(A_i \cap A_j)\)をすべて加えたものを意味するとします.

さて,この予想は正しいのでしょうか??もはやベン図では太刀打ちできません.数学的帰納法で証明してみます.

証明

\(n=l\)のとき予想が正しいとします.この仮定のもとで,\(n=l+1\)のときを考えます.

\[
\begin{align*}
n\left(\displaystyle\bigcup_{i}^{l+1} A_i\right)&=n\left(\left(\displaystyle\bigcup_{i}^{l}A_i\right)\bigcup A_{l+1}\right)\\
&=n\left(\displaystyle\bigcup_{i}^{l}A_i\right)+n\left(A_{l+1}\right)-n\left(\left(\displaystyle\bigcup_{i}^{l}A_i\right) \bigcap A_{l+1}\right)
\end{align*}
\]

ここで,第三項にある\(\left(\displaystyle\bigcup_{i}^{l}A_i\right) \bigcap A_{l+1}\)について考えてみると,

\[
\begin{align*}
&\left(\displaystyle\bigcup_{i}^{l}A_i\right) \bigcap A_{l+1}\\
=&(A_1 \cup A_2 \cup A_3 \cup \cdots \cup A_l)\cap A_{l+1}\\
=&(A_1 \cap A_{l+1} ) \cup (A_2 \cap A_{l+1} ) \cup (A_3 \cap A_{l+1} ) \cup \cdots \cup(A_l \cap A_{l+1} )\\
=&\bigcup^{l}_i(A_i \cap A_{l+1})
\end{align*}
\]

ですから,

\[n\left(\displaystyle\bigcup_{i}^{l+1} A_i\right)=n\left(\displaystyle\bigcup_{i}^{l}A_i\right)+n\left(A_{l+1}\right)-n\left(\bigcup^{l}_i(A_i \cap A_{l+1})\right)\quad \cdots(\ast)\]

を得ます.ここで仮定より,

\[
n\left(\displaystyle\bigcup_{i}^{l} A_i\right)=\displaystyle\sum_{i}^{l} n(A_i)-\displaystyle\sum_{i<j}^{l}n(A_i \cap A_j)+\displaystyle\sum_{i<j<k}^{l}n(A_i \cap A_j \cap A_k)-\cdots+(-1)^{n-1}\displaystyle\sum^{l}_{i<j<\cdots} n(A_i \cap A_j \cap \cdots )
\]

また,この式の\(A_i\)を\(A_i \cap A_{l+1}\)に変えることにより,

\[
\begin{align*}
&n\left(\displaystyle\bigcup_{i}^{l} (A_i \cap A_{l+1})\right)\\
=&\displaystyle\sum_{i}^{l} n(A_i \cap A_{l+1})-\displaystyle\sum_{i<j}^{l}n(A_i \cap A_{l+1} \cap A_j)+\displaystyle\sum_{i<j<k}^{l}n(A_i \cap A_{l+1} \cap A_j \cap A_k)-\cdots+(-1)^{n-1}\displaystyle\sum^{l}_{i<j<\cdots} n(A_i \cap A_{l+1} \cap A_j \cap \cdots )\\
=&\displaystyle\sum_{i}^{l} n(A_i \cap A_{l+1})-\displaystyle\sum_{i<j}^{l}n(A_i \cap A_j \cap A_{l+1})+\displaystyle\sum_{i<j<k}^{l}n(A_i  \cap A_j \cap A_k \cap A_{l+1})-\cdots+(-1)^{n-1}\displaystyle\sum^{l}_{i<j<\cdots} n(A_i  \cap A_j \cap \cdots \cap A_{l+1})
\end{align*}
\]

これら仮定を\((\ast)\)に代入すると,

\[
\begin{align*}
&n\left(\displaystyle\bigcup_{i}^{l+1} A_i\right)\\
=&\displaystyle\sum_{i}^{l} n(A_i)-\displaystyle\sum_{i<j}^{l}n(A_i \cap A_j)+\displaystyle\sum_{i<j<k}^{l}n(A_i \cap A_j \cap A_k)-\cdots+(-1)^{n-1}\displaystyle\sum^{l}_{i<j<\cdots} n(A_i \cap A_j \cap \cdots )\\
&+n\left(A_{l+1}\right)\\
&-\left\{\displaystyle\sum_{i}^{l} n(A_i \cap A_{l+1})-\displaystyle\sum_{i<j}^{l}n(A_i \cap A_j \cap A_{l+1})+\displaystyle\sum_{i<j<k}^{l}n(A_i  \cap A_j \cap A_k \cap A_{l+1})-\cdots+(-1)^{n-1}\displaystyle\sum^{l}_{i<j<\cdots} n(A_i  \cap A_j \cap \cdots \cap A_{l+1})\right\}\\
=&\left\{\displaystyle\sum_{i}^{l} n(A_i)+n\left(A_{l+1}\right)\right\}-\left\{\displaystyle\sum_{i<j}^{l}n(A_i \cap A_j)+\displaystyle\sum_{i}^{l} n(A_i \cap A_{l+1})\right\}+\left\{\displaystyle\sum_{i<j<k}^{l}n(A_i \cap A_j \cap A_k)+\displaystyle\sum_{i<j}^{l}n(A_i \cap A_j \cap A_{l+1})\right\}\\
&-\cdots+(-1)^{n-1}\left\{\displaystyle\sum^{l}_{i<j<\cdots} n(A_i \cap A_j \cap \cdots )+\displaystyle\sum^{l}_{i<j<\cdots} n(A_i  \cap A_j \cap \cdots \cap A_{l+1})\right\}\\
=&\displaystyle\sum_{i}^{l+1} n(A_i)-\displaystyle\sum_{i<j}^{l+1}n(A_i \cap A_j)+\displaystyle\sum_{i<j<k}^{l+1}n(A_i \cap A_j \cap A_k)-\cdots+(-1)^{n-1}\displaystyle\sum^{l+1}_{i<j<\cdots} n(A_i \cap A_j \cap \cdots )
\end{align*}
\]

となって\(n=l+1\)のときも成り立つことが分かります.\(n=1\)のとき成り立つことは自明.(証明終)

見た目はいかついですが高校レベルでも理解できる内容(未習の知識としては\(\bigcup\)の記法と「または,かつ」の分配法則くらい)なので,ぜひ証明にチャレンジしてみてください.集合,場合の数,数学的帰納法,\(\Sigma\)記号の使い方などの総復習にもなりますから.

ちなみに今回証明した式を包除原理と呼びます.

© 2024 佐々木数学塾, All rights reserved.